SlideShare a Scribd company logo
1 of 8
Download to read offline
1586 Arch Pathol Lab Med—Vol 132, October 2008 Pathologic Diagnosis of Gastritis—Sepulveda & Patil
Practical Approach to the Pathologic Diagnosis
of Gastritis
Antonia R. Sepulveda, MD, PhD; Madhavi Patil, MD
● Context.—Most types of gastritis can be diagnosed on
hematoxylin-eosin stains. The most common type of chron-
ic gastritis is Helicobacter pylori gastritis. Reactive or
chemical gastropathy, which is often associated with non-
steroidal anti-inflammatory drug use or bile reflux, is com-
mon in most practices. The diagnosis of atrophic gastritis
can be challenging if few biopsy samples are available and
if the location of the biopsies in the stomach is not known,
such as when random biopsies are sampled in one jar. If
the biopsy site is not known, immunohistochemical stains,
such as a combination of synaptophysin and gastrin, are
useful in establishing the biopsy location.
Objective.—To demonstrate a practical approach to
achieving a pathologic diagnosis of gastritis by evaluating
a limited number of features in mucosal biopsies.
Data Source.—In this article, we present several repre-
sentative gastric biopsy cases from a gastrointestinal pa-
thology practice to demonstrate the practical application
of basic histopathologic methods for the diagnosis of gas-
tritis.
Conclusions.—Limited ancillary tests are usually re-
quired for a diagnosis of gastritis. In some cases, special
stains, such as acid-fast stains, and immunohistochemical
stains, such as for H pylori and viruses, can be useful. Hel-
icobacter pylori immunohistochemical stains can particu-
larly contribute (1) when moderate to severe, chronic gas-
tritis or active gastritis is present but no Helicobacter or-
ganisms are identified upon hematoxylin-eosin stain; (2)
when extensive intestinal metaplasia is present; and (3) in
follow-up biopsies, after antibiotic treatment for H pylori.
(Arch Pathol Lab Med. 2008;132:1586–1593)
Gastritis refers to a group of diseases characterized by
inflammation of the gastric mucosa. Histologic ex-
amination of gastric mucosal biopsies is necessary to es-
tablish a diagnosis of gastritis. In clinical practice, the role
of the pathologist who evaluates a gastric biopsy for gas-
tritis is to find the cause of gastritis because that will pro-
vide direct targets toward which therapeutic measures can
be directed. An etiologic classification of gastritis is pre-
sented at the end of this section. Comprehensive reviews
of gastritis have been published.1,2,3
The goal of this article
is to present a practical approach to the diagnosis of the
most common types of gastritis encountered in a large
practice of gastrointestinal pathology. The reader will be
presented several cases representative of typical forms of
gastritis; for each case, the reader will be prompted
through a series of questions to examine the histologic
features of the mucosa, leading to a pattern of answers
and to a final diagnosis.
The first question is aimed at determining whether or
not there are features of chronic or acute (active) gastritis
Accepted for publication January 10, 2008.
From the Department of Pathology and Laboratory Medicine, Hos-
pital of the University of Pennsylvania, Philadelphia.
The authors have no relevant financial interest in the products or
companies described in this article.
Presented in part at the 47th Annual Meeting of the Houston Society
of Clinical Pathologists, Houston, Tex, April 21, 2007.
Reprints: Antonia R. Sepulveda, MD, PhD, Department of Pathology
and Laboratory Medicine, University of Pennsylvania, 3400 Spruce St,
Founders Six, Philadelphia, PA 19104 (e-mail: asepu@mail.med.
upenn.edu).
present. If the biopsy shows chronic gastritis, the follow-
ing questions should be posed:
1. Are there features of chronic gastritis present? Lym-
phocytic and plasmacytic inflammatory reaction indicates
chronic gastritis.
2. Are there neutrophils in the mucosa? The presence
of neutrophils indicate active gastritis.
3. Is there Helicobacter?
4. Is there glandular atrophy? Is intestinal metaplasia
present?
5. What is the topography of lesions (predominantly in
the oxyntic mucosa of the body and fundus, predomi-
nantly in antrum, or involving both locations)?
6. Are there special features (such as granulomas, fo-
veolar hyperplasia, viral inclusions)?
7. What ancillary studies are indicated, and what are
the results?
TYPES OF CHRONIC GASTRITIS
Infectious Gastritis
Helicobacter pylori infection is the most common cause of
chronic gastritis. Other forms of infectious gastritis include
the following: Helicobacter heilmannii–associated gastritis;
granulomatous gastritis associated with gastric infections
in mycobacteriosis, syphilis, histoplasmosis, mucormyco-
sis, South American blastomycosis, anisakiasis or anisak-
idosis; chronic gastritis associated with parasitic infec-
tions; and viral infections, such as cytomegalovirus and
herpesvirus infection.
Arch Pathol Lab Med—Vol 132, October 2008 Pathologic Diagnosis of Gastritis—Sepulveda & Patil 1587
Figure 1. Helicobacter pylori–associated chronic active gastritis. A,
Chronic inflammation oriented toward the surface of the mucosa. Neu-
trophils cannot be seen at this magnification (original magnification
⫻10) but were identified with high power (hematoxylin-eosin stain). B,
Circled area shows H pylori organisms within the mucus layer close
to the surface of gastric epithelial cells (hematoxylin-eosin, original
magnification ⫻40). C, Gastric mucosa with intestinal metaplasia (he-
matoxylin-eosin, original magnification ⫻20).
Noninfectious Gastritis
Noninfectious gastritis is associated with autoimmune
gastritis; reactive or chemical gastropathy, usually related
to chronic bile reflux or nonsteroidal anti-inflammatory
drug (NSAID) intake; uremic gastropathy; noninfectious
granulomatous gastritis; lymphocytic gastritis, including
gastritis associated with celiac disease; eosinophilic gas-
tritis; radiation injury to the stomach; graft-versus-host
disease; ischemic gastritis; and gastritis secondary to che-
motherapy.
Many cases of gastritis are of undetermined cause and
present as chronic, inactive gastritis with various degrees
of severity.3
TYPES OF ACUTE GASTRITIS
Many of the forms of chronic gastritis may present with
an acute form, with progression to chronic gastritis be-
cause of persisting injury or sequelae. This is the case of
gastritis associated with long-term intake of aspirin and
other NSAIDs and bile reflux into the stomach; excessive
alcohol consumption; heavy smoking; cancer chemother-
apeutic drugs and radiation; acids and alkali in suicide
attempts; uremia; severe stress (trauma, burns, surgery);
ischemia and shock; systemic infections; mechanical trau-
ma, such as intubation associated mucosal lesions; and vi-
ral infections.
Case 1
A 60-year-old man underwent esophagogastroduoden-
oscopy. A biopsy of gastric antrum was submitted to pa-
thology to rule out H pylori. The histologic findings are
shown in Figure 1, A through C.
Findings. Examination of the biopsy material available
gives the following answers:
1. Are there features of chronic gastritis? Yes. The gas-
tric antral mucosa shows expansion of the lamina propria
by chronic inflammatory cells, consisting of plasma cells
and small lymphocytes, predominantly located toward the
luminal aspect of the mucosa, a pattern that is suggestive
of H pylori infection.
2. Are there neutrophils in the mucosa? Yes. Therefore,
this represents active gastritis. This is a mild form of active
gastritis.
3. Is there Helicobacter? Yes. Hematoxylin-eosin (H&E)
examination reveals diagnostic H pylori bacterial forms in
the surface mucus layer in close proximity to the apical
aspect of surface epithelial cells.
4. Is there glandular atrophy? The biopsy sample avail-
able is not adequate for evaluation of atrophic gastritis;
multiple biopsies, including samples of gastric body, are
necessary for adequate evaluation of glandular atrophy. Is
there intestinal metaplasia? Yes.
5. What is the topography of lesions? The chronic gas-
tritis in this case involves, at minimum, the gastric antrum;
it is advisable to obtain biopsy samples of both gastric
antrum and body for a better evaluation of gastritis, as
recommended by the updated Sydney guidelines4
for clas-
sification of gastritis.
6. Are additional special features present? No.
7. Are special stains recommended? No.
Diagnosis. Gastric antral mucosa with H pylori–asso-
ciated chronic gastritis, mildly active, and focal intestinal
metaplasia.
H PYLORI–ASSOCIATED CHRONIC GASTRITIS
The Helicobacter species consist of gram-negative rods
that infect the gastric mucosa. Helicobacter pylori bacteria
are 3.5 ␮m long and are generally comma-shaped or have
slightly spiral forms. Helicobacter heilmannii, a rare agent
of chronic gastritis, is a 5- to 9-␮m-long bacterium, with
a characteristic tightly corkscrew-shaped, spiral form.5
Helicobacter pylori infection usually is acquired during
childhood, persisting as chronic gastritis if the organism
is not eradicated. During progression of gastritis over the
years, the gastric mucosa undergoes a sequence of changes
that may lead to glandular atrophy, intestinal metaplasia,
increased risk of gastric dysplasia and carcinoma,6–9
and
mucosa-associated lymphoid tissue lymphoma,10,11
report-
ed as extranodal, marginal zone, B-cell lymphoma in the
World Health Organization classification.12
Helicobacter pylori infection is associated with the histo-
logic pattern of active and chronic gastritis, reflecting the
presence of neutrophils and mononuclear cells (lympho-
cytes and plasma cells) in the mucosa, respectively. The
term active gastritis is preferred to acute gastritis because H
pylori gastritis is a long-standing chronic infection with
ongoing activity. Lymphoid aggregates and lymphoid fol-
licles may be observed expanding the lamina propria, and
rare lymphocytes may enter the epithelium. Helicobacter
pylori organisms are found within the gastric mucus layer
that overlays the apical side of gastric surface cells, and
lower numbers are found in the lower portions of the gas-
tric foveolae. Helicobacter pylori may be found within the
deeper areas of the mucosa in association with glandular
cells in patients on acid blockers, such as the commonly
used proton pump inhibitors.13
Helicobacter pylori–associated gastritis can display differ-
ent levels of severity. The severity of H pylori gastritis ac-
tivity may be indicated in a pathology report as mild (rare
neutrophils seen), moderate (obvious neutrophils within
the glandular and foveolar epithelium), or severe (numer-
ous neutrophils with glandular microabscesses and mu-
cosal erosion or frank ulceration).4,14
Helicobacter pylori–associated chronic gastritis can mani-
fest as a pangastritis involving the area from the pylorus
to the gastric body and cardia, or it may predominantly
involve the antrum. Patients with gastric ulcers generally
have antral-predominant gastritis, whereas pangastritis,
1588 Arch Pathol Lab Med—Vol 132, October 2008 Pathologic Diagnosis of Gastritis—Sepulveda & Patil
Figure 2. Chronic active gastritis in a patient with Crohn disease. A
glandular abscess is shown; additionally, there are many neutrophils in
the lamina propria admixed with a background of chronic inflamma-
tion (hematoxylin-eosin, original magnification ⫻20).
or at least multifocal gastritis, is more common in patients
with gastric carcinoma. The latter generally have signifi-
cant intestinal metaplasia and gastric oxyntic glandular
atrophy coexisting in the background stomach. It is im-
portant to make a pathologic diagnosis of atrophic gastri-
tis because gastric atrophy is associated with increased
risk of gastric cancer.15,16
Patients with chronic atrophic
gastritis may have up to a 16-fold increased risk of devel-
oping gastric carcinoma, compared with the general pop-
ulation.15,17
When large numbers of H pylori are present in the mu-
cosa, the identification of typical organisms is generally
possible on H&E stains. However, there are cases of chron-
ic, active gastritis with features suggestive of H pylori gas-
tritis in which the organisms are not detected. Several spe-
cial stains have been extensively used to help identify H
pylori organisms in the gastric mucosa, including modi-
fied-Giemsa, Genta, thiazine stains, and immunohisto-
chemistry against Helicobacter antigens. The selection of the
special stain used is largely dependent on preferences re-
lated to individual practices. Although, overall, no major
differences in sensitivity and specificity have been report-
ed, studies have recommended immunohistochemical
stains in a subset of cases.18,19
In our practice, we prefer to
use immunohistochemical stains for detection of H pylori
if organisms are not found on H&E stains in the following
cases: (1) if moderate to severe chronic gastritis or any
grade of active gastritis is present but no Helicobacter or-
ganisms are identified on H&E; (2) when extensive intes-
tinal metaplasia is present because H pylori density is re-
duced in areas of intestinal metaplasia; and (3) during
follow-up biopsies after antibiotic treatment for H pylori.
Helicobacter heilmannii may cause similar pathology, and
the treatment is similar to H pylori.5
Case 2
A 45-year-old man is seen to rule out H pylori. He pre-
sents with a history of Crohn disease. The histologic find-
ings are shown in Figure 2.
Findings. Examination of the biopsy material results
in the following pattern of answers:
1. Are there features of chronic gastritis? Yes. The gas-
tric antral mucosa shows expansion of the lamina propria
by chronic inflammatory cells, consisting of admixed plas-
ma cells and small lymphocytes, throughout the thickness
of the mucosa.
2. Are there neutrophils in the mucosa? Yes, with an
occasional glandular abscess; therefore, there is active gas-
tritis. Of note, the active gastritis has a patchy distribution.
3. Is there Helicobacter? No. Examination with H&E
stain does not reveal such bacterial forms. Immunohisto-
chemical stain is performed.
4. Is there atrophy? The biopsy sample available is not
adequate for evaluation of atrophic gastritis because the
biopsy material is only from the gastric antrum; multiple
gastric body biopsies are necessary for adequate evalua-
tion of glandular atrophy. There is no intestinal metapla-
sia.
5. What is the topography of lesions? The chronic gas-
tritis involves, at minimum, the gastric antrum.
6. Are additional special features seen? No. Although
in a case of Crohn disease gastritis, epithelioid granulo-
mas may be present; in this case, no granulomas were
seen.
7. Are special stains recommended? Yes. Helicobacter py-
lori immunohistochemical stain, which is helpful in cases
where Crohn disease is suspected because the absence of
H pylori organisms in chronic active gastritis is consistent
with Crohn disease. The H pylori immunohistochemical
stain in this case is negative.
Diagnosis. Gastric antral mucosa with chronic active
gastritis, moderately active, patchy. No H pylori organisms
are identified by H&E or immunohistochemistry. Note:
These features are consistent with Crohn disease–associ-
ated gastritis.
CROHN DISEASE–ASSOCIATED GASTRITIS
The hallmark histopathologic features of Crohn disease–
associated gastritis are the presence of patchy, acute in-
flammation with possible gastric pit or glandular abscess-
es, commonly with a background with lymphoid aggre-
gates. Recent studies20
reported the presentation of gastri-
tis in patients with Crohn disease as a focally enhanced
gastritis, characterized by small collections of lymphocytes
and histiocytes surrounding a small group of gastric fo-
veolae or glands, often with infiltrates of neutrophils. In
severe cases, there may be diffuse inflammation in the
lamina propria, with variable glandular loss, fissures, ul-
cers, transmural inflammation, and fibrosis. Noncaseating
epithelioid granulomas may be present in about one third
of cases of Crohn disease gastritis but are often not seen,
at least in part, because of limited tissue sampling.
When granulomas are identified, the differential diag-
nosis includes other forms of granulomatous gastritis.
There are infectious and noninfectious causes of granulo-
matous gastritis. Noninfectious diseases represent the usu-
al cause of gastric granulomas and include Crohn disease,
sarcoidosis, and isolated granulomatous gastritis. Sarcoid-
like granulomas may be observed in cocaine users, and
foreign material is occasionally observed in the granulo-
mas. Sarcoidosis of the stomach is usually associated with
granulomas in other organs, especially the lungs, hilar
nodes, or salivary glands. A diagnosis of idiopathic, iso-
Arch Pathol Lab Med—Vol 132, October 2008 Pathologic Diagnosis of Gastritis—Sepulveda & Patil 1589
Figure 3. Helicobacter pylori–associated atrophic gastritis. A, Chronic
active gastritis involving the gastric oxyntic mucosa with glandular at-
rophy (hematoxylin-eosin, original magnification ⫻10). Inset shows
rare neutrophils into the glandular epithelium (white arrows). B, Im-
munohistochemical stain for H pylori shows a small area with organ-
isms attached to the surface epithelium. Insets show individual Heli-
cobacter bacteria (thin arrows) with characteristic elongated, slightly
spiral S shape or clusters of packed bacteria (thick arrowhead) closely
adherent to the surface of epithelial cells. Immunohistochemical stains
are useful in cases such as this one, when H pylori organisms are
closely associated with the surface epithelial cells making it difficult to
ascertain the characteristic bacterial morphology on hematoxylin-eosin
stains (original magnification ⫻40).
lated, granulomatous gastritis is rendered when known
entities associated with granulomas are excluded.
Case 3
A 60-year-old man presents with a nodularity of the
gastric body to rule out H pylori. Esophagogastroduoden-
oscopy with biopsy of the nodular areas was performed.
The histologic findings are shown in Figure 3, A and B.
Findings. Examination of the biopsy material results
in the following pattern of answers:
1. Are there features of chronic gastritis? Yes.
2. Are there neutrophils in the mucosa? Yes. There are
neutrophils in the mucosa, representing active gastritis.
3. Is there Helicobacter? No. Examination with H&E
stain does not reveal H pylori bacterial forms. Immunohis-
tochemical is performed.
4. Is there atrophy? Yes. There is a reduced number of
oxyntic glands in the biopsy. There is no intestinal meta-
plasia.
5. What is the topography of lesions? The chronic gas-
tritis involves, at minimum, the gastric body.
6. Are additional special features seen? No.
7. Are special stains recommended? Yes. Helicobacter py-
lori immunohistochemical stain, which is positive.
Diagnosis. Gastric oxyntic mucosa with H pylori–as-
sociated chronic active gastritis and glandular atrophy,
moderate. No intestinal metaplasia is identified. Helicobac-
ter organisms are identified by immunohistochemistry.
ATROPHIC GASTRITIS
Several publications, including those reporting the Syd-
ney system and the updated Houston classification of gas-
tritis, have proposed criteria for the evaluation of atrophic
gastritis. Interobserver variability is significant, especially
in the evaluation of antral atrophy.4,21
Recent advances that
appear to decrease the interobserver variation in the as-
sessment of gastric atrophy have been reported.14
Atrophy
is more accurately assessed after resolution of severe in-
flammation of the mucosa; therefore, if there is H pylori
gastritis, the infection should be eradicated before atrophy
is difinitively evaluated. When marked inflammation is
present, a diagnosis of indefinite for atrophy may be of-
fered, especially if there is no intestinal metaplasia.
The recommended definition of atrophy is the loss of
appropriate glands, and atrophy can be scored according
to the degree of severity as mild, moderate, or severe.22
In
this definition, intestinal metaplasia represents a form of
atrophy described as metaplastic atrophy (or gastric glan-
dular atrophy with intestinal metaplasia).
Gastric atrophy is usually associated with intestinal
metaplasia. However, in limited endoscopic biopsies, in-
testinal metaplasia might not be sampled, whereas the
mucosa shows definitive atrophy. Usually gastric atrophy
and intestinal metaplasia occur on a background of chron-
ic gastritis, hence the term atrophic gastritis.
Sampling of the mucosa for evaluation of atrophy and
gastritis is generally adequate by using the 5 biopsies rec-
ommended by the Sydney system, including 2 biopsies
from the antrum, 2 from the corpus or body, and 1 from
the incisura angularis.4,21
It is essential for the pathologist
to have a means of determining the specific site in the
stomach where a biopsy is sampled from because specific
topography of atrophy characterizes the different types of
atrophic gastritis. In atrophic gastritis associated with H
pylori, glandular atrophy and intestinal metaplasia involve
both the gastric antrum and body, whereas in autoim-
mune atrophic gastritis, the disease is essentially restricted
to the gastric body. Ideally, the precise location is indicated
by the endoscopist, and the biopsies from different sites
are submitted in separate containers. However, using spe-
cial stains can help the pathologist determine the location
of the biopsy fragments received. This approach is ex-
emplified in case 5.
Gastric atrophy and intestinal metaplasia are associated
with increased gastric cancer risk, but unlike the intestinal
metaplasia of Barrett syndrome, no specific recommen-
dations for surveillance have been established in the Unit-
ed States, although published data in other populations
have suggested a benefit.23
In that study,23
patients with
1590 Arch Pathol Lab Med—Vol 132, October 2008 Pathologic Diagnosis of Gastritis—Sepulveda & Patil
Figure 4. Autoimmune gastritis. A, The gastric biopsy location in
stomach was not provided in this case (hematoxylin-eosin, original
magnification ⫻20). B, Immunohistochemical stain for synaptophysin
demonstrates enterochromaffin-like cell hyperplasia (original magnifi-
cation ⫻20). The inset shows individual enterochromaffin-like cells
staining brown, indicated by the arrow. C, Immunohistochemical stain
for gastrin is negative, indicating the biopsy site to be from the gastric
oxyntic mucosa (original magnification ⫻20).
extensive atrophic gastritis and intestinal metaplasia had
an 11% risk of gastric malignancy.
Case 4
Esophagogastroduodenoscopy of a 60-year-old man
shows gastritis. The pathologist needs to rule out H pylori
and gastric atrophy. The gastric site of the biopsy is not
specified. Figure 4, A through C, represents the histologic
findings.
Findings. Examination of the biopsy material results
in the following pattern of answers:
1. Are there features of chronic gastritis? Yes.
2. Are there neutrophils in the mucosa? Yes. There are
neutrophils in the mucosa; therefore, there is a component
of active gastritis.
3. Is there Helicobacter? No. Examination with H&E
stains do not reveal H pylori bacterial forms. Immunohis-
tochemical stain is performed.
4. Is there atrophy? If the biopsy is from gastric oxyntic
mucosa then there is atrophy, however, if the specimen is
from the antrum, it may represent chronic gastritis with-
out atrophy. There is no intestinal metaplasia.
5. Are special stains recommended? Yes. Immunohis-
tochemical stains for synaptophysin and gastrin are per-
formed. Immunohistochemical stains for synaptophysin
(Figure 4, B), show a linear pattern of synaptophysin-pos-
itive cells, whereas the gastrin stain is negative. Because
gastrin is negative, the biopsy is not from the gastric an-
trum (G cells are characteristically located in the antrum
and pylorus), and therefore, it can be established that the
biopsy is of oxyntic mucosa with reduced oxyntic glan-
dular profiles, establishing a diagnosis of atrophy. The lin-
ear arrays of synaptophysin-positive cells represent en-
terochromaffin-like cell hyperplasia. Enterochromaffin-
like cell hyperplasia occurs in response to hypergastri-
nemia that results from hypochlorhydria associated with
gastric oxyntic cell atrophy.
6. Are additional special features seen? No.
7. Is immunohistochemical stain for H pylori positive?
No.
Diagnosis. Gastric oxyntic mucosa with chronic active
gastritis and glandular atrophy, severe. No intestinal meta-
plasia is identified. No Helicobacter organisms are identi-
fied. Note: These features are most suggestive of autoim-
mune gastritis.
AUTOIMMUNE ATROPHIC GASTRITIS
This form of gastritis (reviewed in Sepulveda et al1
and
Capella et al24
) is caused by antiparietal cell and anti-in-
trinsic factor antibodies and presents as a chronic gastritis
with oxyntic cell injury, and glandular atrophy essentially
restricted to the oxyntic mucosa of the gastric body and
fundus. The histologic changes vary in different phases of
the disease. During the early phase, there is multifocal
infiltration of the lamina propria by mononuclear cells and
eosinophils and focal T-cell lymphocyte infiltration of ox-
yntic glands with glandular destruction. Focal mucous
neck cell hyperplasia (pseudopyloric metaplasia), and hy-
pertrophic changes of parietal cells are also observed.
During the florid phase, there is increased lymphocytic
inflammation, oxyntic gland atrophy, and focal intestinal
metaplasia. The end stage is characterized by diffuse in-
volvement of the gastric body and fundus by chronic atro-
phic gastritis associated with multifocal intestinal meta-
plasia. In contrast to the gastric body, the antrum is
spared. Recently, a distinct form of autoimmune gastritis,
characterized by atrophic pangastritis, was reported in a
small group of patients with systemic autoimmune dis-
orders.25
Autoimmune gastritis is a relatively rare disease but
represents the most frequent cause of pernicious anemia
in temperate climates. The risk of gastric adenocarcinoma
was reported to be at least 2.9 times higher in patients
with pernicious anemia than in the general population,
and there is also an increased risk of gastric carcinoid tu-
mors.
Case 5
A 47-year-old woman presents with a history of celiac
disease. Esophagogastroduodenoscopy was performed,
with biopsy of gastric antrum. The pathologist needs to
rule out H pylori. Figure 5, A and B, illustrates the histo-
logic findings.
Findings. Examination of the biopsy material results
in the following pattern of answers:
1. Are there features of chronic gastritis? Yes. There are
large numbers of intraepithelial lymphocytes.
2. Are there neutrophils in the mucosa? No.
3. Is there Helicobacter? No. Examination with H&E
Arch Pathol Lab Med—Vol 132, October 2008 Pathologic Diagnosis of Gastritis—Sepulveda & Patil 1591
Figure 5. Lymphocytic gastritis. A, Gastric mucosal surface epithelium
is studded with intraepithelial small lymphocytes (hematoxylin-eosin,
original magnification ⫻20). B, Immunohistochemistry highlights nu-
merous CD3-positive T lymphocytes staining dark brown (original mag-
nification ⫻20). The lymphocytes predominantly infiltrate the surface
and foveolar epithelium. The thick arrow points to light-brown back-
ground stain found in some glandular cells. The inset shows a higher
magnification of the surface epithelium containing many intraepithelial
lymphocytes (original magnification ⫻40). One individual lymphocyte
is indicated by the arrow. T lymphocytes stain dark brown, whereas
the nucleus of the epithelial cell nuclei stain with the blue counterstain.
stain does not reveal H pylori bacterial forms. Immunohis-
tochemical is performed.
4. Is there atrophy? No. There is no glandular atrophy
and no intestinal metaplasia.
5. What is the topography of lesions? The chronic gas-
tritis involves, at minimum, the gastric antrum.
6. Are additional special features seen? Yes. The specific
features in this biopsy include a characteristic intraepithe-
lial lymphocytosis. Immunohistochemical stain for CD3 is
positive, highlighting a population of T lymphocytes in
the mucosa and, typically, many intraepithelial lympho-
cytes.
7. Are special stains recommended? Yes. Immunohis-
tochemical stain for H pylori, which is negative.
Diagnosis. Chronic gastritis with increased intraepi-
thelial T lymphocytes. No Helicobacter organisms are iden-
tified. Note: These features are consistent with lympho-
cytic gastritis–associated with celiac disease.
LYMPHOCYTIC GASTRITIS
Lymphocytic gastritis is a type of chronic gastritis char-
acterized by marked infiltration of the gastric surface and
foveolar epithelium by T lymphocytes and by chronic in-
flammation in the lamina propria. A diagnosis can be ren-
dered when 30 or more lymphocytes per 100 consecutive
epithelial cells are observed, and the counts are recom-
mended in biopsies from the gastric corpus. The endo-
scopic pattern is, in some cases, described as varioliform
gastritis. The cause of lymphocytic gastritis is usually un-
known, but some cases are seen in patients with gluten-
sensitive enteropathy/celiac disease and in Ménétrier disease.
Smaller numbers of intraepithelial lymphocytes can also
be seen in H pylori gastritis, but the diagnosis of lympho-
cytic gastritis should be reserved for cases with marked
intraepithelial lymphocytosis in the absence of active H
pylori gastritis. Lymphocytic gastritis can be observed in
children but is usually detected in late adulthood, with
average age of diagnosis of 50 years.
Case 6
A 75-year-old woman presents after esophagogastro-
duodenoscopy. Gastric antrum shows gastritis; the pa-
thologist is asked to rule out H pylori. The histologic find-
ings are shown in Figure 6.
Findings. Examination of the biopsy material results
in the following pattern of answers:
1. Are there features of chronic gastritis? There is min-
imal chronic gastritis.
2. Are there neutrophils in the mucosa? No.
3. Is there Helicobacter? No. Examination of H&E stains
does not reveal H pylori bacterial forms.
4. Is there atrophy? No. There is no atrophy or intestinal
metaplasia.
5. What is the topography of lesions? The chronic gas-
tritis involves, at minimum, the gastric antrum.
6. Are additional special features seen? Yes. There are
diagnostic special features, including foveolar hyperplasia
with a corkscrew appearance of the foveolae. The foveolar
epithelium shows reactive cytologic features, including re-
duced cytoplasmic mucin. The lamina propria shows con-
gestion and smooth muscle hyperplasia, with prominent
muscularization of the most superficial mucosa.
7. Are special stains recommended? No ancillary tests
are performed.
Diagnosis. Gastric antral mucosa with features con-
sistent with reactive gastropathy. No H pylori organisms
are identified.
CHRONIC, REACTIVE (CHEMICAL) GASTROPATHY
Chronic reactive gastropathy (also know as chemical
gastropathy) is very common in current clinical practice.
The mucosal changes are usually more prominent in the
prepyloric region, but they may extend to involve the ox-
yntic mucosa. The usual underlying causes include chron-
ic bile reflux and long-term NSAID intake. The histopath-
ologic features include mucosal edema, congestion, fibro-
muscular hyperplasia in the lamina propria, and foveolar
hyperplasia with a corkscrew appearance in the most se-
vere forms. The foveolar epithelium characteristically
shows reactive nuclear features and reduction of mucin.
The epithelial changes occur with little background chron-
ic inflammation. However, if there is erosion of the mu-
cosa, superficial neutrophils may be present. Erosive gas-
1592 Arch Pathol Lab Med—Vol 132, October 2008 Pathologic Diagnosis of Gastritis—Sepulveda & Patil
Figure 6. Reactive gastropathy (hematoxylin-eosin, original magnifi-
cation ⫻20).
Figure 7. Erosive gastritis and cytomegalovirus-associated gastritis. A,
Gastric mucosa with erosion in a patient with history of nonsteroidal
anti-inflammatory drug use (hematoxylin-eosin, original magnification
⫻20). B, Gastric mucosal erosion with granulation tissue. Inset shows
←
cytomegalovirus inclusion. This single inclusion is identified by the ar-
row on the background granulation tissue (hematoxylin-eosin, original
magnifications ⫻20 and ⫻40 [inset]).
tritis (Figure 7, A) can present clinically as acute gastritis,
often associated with NSAID intake.
The features associated with bile reflux are typically
found in patients with partial gastrectomy, in whom, the
lesions develop near the surgical stoma. However, alter-
ations induced by bile reflux also affect the intact stomach.
A recent study26
reported altered mucin expression in re-
active gastropathy, including aberrant expression of
MUC5Ac in pyloric glands. Evaluation of mucin-expres-
sion patterns can be useful to support a diagnosis of re-
active gastropathy; however, additional studies are war-
ranted to validate this potential application of mucin im-
munohistochemistry.
Case 7
A 45-year-old woman presents with a history of bone
marrow transplant. Esophagogastroduodenoscopy shows
gastric erosion. The histologic findings are represented in
Figure 7, B.
Findings. Examination of the biopsy material results
in the following pattern of answers:
1. Are there features of chronic gastritis? Yes. The sam-
ple of gastric mucosa reveals mucosal erosion with gran-
ulation tissue and associated chronic and acute inflam-
mation.
2. Are there neutrophils in the mucosa? Yes. There are
superficial neutrophils in the mucosa, but they are limited
to the area of mucosal erosion.
3. Is there Helicobacter? No. Examination with H&E
stain does not reveal such bacterial forms.
4. Is there atrophy? No. There is no atrophy or intestinal
metaplasia.
5. What is the topography of lesions? Away from the
areas of erosion, there is no evidence of gastritis; therefore,
the location of the biopsy is not contributory in this case.
6. Are additional special features seen? Yes. There are
special features including enlarged cells, arousing suspi-
cion of cytomegalovirus inclusions in the granulation tis-
sue.
7. Are special stains recommended? Yes. Immunohis-
tochemical stain for cytomegalovirus reveals rare but char-
acteristic viral inclusions (not shown).
Diagnosis. Gastric antral mucosa with erosion and cy-
tomegalovirus inclusions, consistent with cytomegalovirus-
associated gastritis.
CYTOMEGALOVIRUS GASTRITIS
Cytomegalovirus infection of the stomach is observed
in patients with underlying immunosuppression. Histo-
logically, intranuclear eosinophilic inclusions and smaller
intracytoplasmic inclusions in enlarged cells are charac-
teristic. A patchy, mild inflammatory infiltrate is observed
in the lamina propria. Viral inclusions are present in en-
dothelial or mesenchymal cells in the lamina propria and
may be seen in gastric epithelial cells. Severe activity may
result in mucosal ulceration.
Arch Pathol Lab Med—Vol 132, October 2008 Pathologic Diagnosis of Gastritis—Sepulveda & Patil 1593
COMMENT
Most types of gastritis can be diagnosed with H&E
stains. To reach a determination of etiology and a specific
diagnostic entity, a limited list of questions can be used
to evaluate the histopathology of gastric biopsies, which
can lead to a pattern of answers that corresponds to a
specific diagnosis of the most common types of gastritis.
Although not ideal, the diagnosis of gastritis can be
reached from limited biopsy material, even when the lo-
cation of the biopsy is not indicated. If the biopsy site is
not known, immunohistochemical stains for synaptophy-
sin and gastrin can help determine the biopsy location,
permitting a specific diagnosis of atrophic gastritis type.
Helicobacter pylori immunohistochemical stains can be par-
ticularly useful when moderate to severe chronic gastritis
or any active gastritis is present but no Helicobacter organ-
isms are identified on H&E stains, when extensive intes-
tinal metaplasia is present, and to evaluate follow-up bi-
opsies after antibiotic treatment for H pylori.
At the end of the day, there are a number of cases with
a diagnosis of chronic inactive gastritis, generally mild, for
which a specific etiology cannot be determined by histo-
pathologic examination alone. This may be accounted for
by limited tissue sampling, nonspecific focal, mild, chronic
inactive gastritis associated with various systemic disor-
ders, or as yet uncharacterized forms of gastritis.
References
1. Sepulveda AR, Dore MP, Bazzoli F. Chronic gastritis. Available at: http://
www.emedicine.com. Accessed November 27, 2007.
2. Srivastava A, Lauwers GY. Pathology of non-infective gastritis. Histopathol-
ogy. 2007;50:15–29.
3. McKenna BJ, Appelman HD. Primer: histopathology for the clinician—how
to interpret biopsy information for gastritis. Nat Clin Pract Gastroenterol Hepatol.
2006;3:165–171.
4. Dixon MF, Genta RM, Yardley JH, Correa P, the participants in the Inter-
national Workshop on the Histopathology of Gastritis, Houston 1994. Classifi-
cation and grading of gastritis: the updated Sydney System. Am J Surg Pathol.
1996;20:1161–1181.
5. Singhal AV, Sepulveda AR. Helicobacter heilmannii gastritis: a case study
with review of literature. Am J Surg Pathol. 2005;29:1537–1539.
6. Sipponen P, Kekki M, Haapakoski J, Ihamaki T, Siurala M. Gastric cancer
risk in chronic atrophic gastritis: statistical calculations of cross-sectional data. Int
J Cancer. 1985;35:173–177.
7. Uemura N, Okamoto S, Yamamoto S, et al. Helicobacter pylori infection
and the development of gastric cancer. N Engl J Med. 2001;345:784–789.
8. Asaka M, Sugiyama T, Nobuta A. et al. Atrophic gastritis and intestinal meta-
plasia in Japan: results of a large multicenter study: Helicobacter. 2001;6:294–
299.
9. Correa P, Haenszel W, Cuello C, Tannenbaum S, Archer M. A model for
gastric cancer epidemiology. Lancet. 1975;2:58–60.
10. Parsonnet J, Hansen S, Rodriguez L, et al. Helicobacter pylori infection
and gastric lymphoma. N Engl J Med. 1994;330:1267–1271.
11. Wotherspoon AC, Ortiz-Hidalgo C, Falzon MR, Isaacson PG. Helicobacter
pylori–associated gastritis and primary B-cell gastric lymphoma. Lancet. 1991;
338:1175–1176.
12. Jaffe ES, Harris NL, Stein H, Vardiman JW. Pathology and Genetics of Tu-
mours of Hematopoietic and Lymphoid Tissues. Lyon, France: IARC Press; 2001.
World Health Organization Classification of Tumours; vol 3.
13. Tagkalidis P, Royce S, Macrae F, Bhathal P. Selective colonization by Hel-
icobacter pylori of the deep gastric glands and intracellular canaliculi of parietal
cells in the setting of chronic proton pump inhibitor use. Eur J Gastroenterol
Hepatol. 2002;14:453–456.
14. Rugge M, Genta RM. Staging and grading of chronic gastritis. Hum Pathol.
2005;36:228–233.
15. Huang JQ, Sridhar S, Chen Y, Hunt RH. Meta-analysis of the relationship
between Helicobacter pylori seropositivity and gastric cancer. Gastroenterology.
1998;114:1169–1179.
16. Correa P, Houghton J. Carcinogenesis of Helicobacter pylori. Gastroenter-
ology. 2007;133:659–72.
17. Sepulveda AR, Coelho LG. Helicobacter pylori and gastric malignancies.
Helicobacter. 2002;7(suppl 1):37–42.
18. Jonkers D, Stobberingh E, de Bruine A, Arends JW, Stockbrugger R. Eval-
uation of immunohistochemistry for the detection of Helicobacter pylori in gastric
mucosal biopsies. J Infect. 1997;35:149–154.
19. Toulaymat M, Marconi S, Garb J, Otis C, Nash S. Endoscopic biopsy pa-
thology of Helicobacter pylori gastritis: comparison of bacterial detection by im-
munohistochemistry and Genta stain. Arch Pathol Lab Med. 1999;123:778–781.
20. Xin W, Greenson JK. The clinical significance of focally enhanced gastritis.
Am J Surg Pathol. 2004;28:1347–1351.
21. Price A. The Sydney System: histological division. J Gastroenterol Hepatol.
1991;6:209–222.
22. Rugge M, Correa P, Dixon MF. et al. Gastric mucosal atrophy: interob-
server consistency using new criteria for classification and grading. Aliment Phar-
macol Ther. 2002;16:1249–1259.
23. Whiting JL, Sigurdsson A, Rowlands DC, Hallissey MT, Fielding JW. The
long term results of endoscopic surveillance of premalignant gastric lesions. Gut.
2002;50:378–381.
24. Capella R, Fiocca C, Cornaggia M. Autoimmune gastritis. In: Graham DY,
Genta RM, Dixon MF, eds. Gastritis. Philadelphia, Pa: Lippincott Williams; 1999:
79–96.
25. Jevremovic D, Torbenson M, Murray JA, Burgart LJ, Abraham SC. Atrophic
autoimmune pangastritis: a distinctive form of antral and fundic gastritis associ-
ated with systemic autoimmune disease. Am J Surg Pathol. 2006;30:1412–1419.
26. Mino-Kenudson M, Tomita S, Lauwers GY. Mucin expression in reactive
gastropathy: an immunohistochemical analysis. Arch Pathol Lab Med. 2007;131:
86–90.
Archives of Pathology & Laboratory Medicine and Archives of Ophthalmology will publish a joint
theme issue on ophthalmic pathology in August 2009. Articles on diagnostic procedures, path-
ologic mechanistic pathways, and translational research in retinoblastoma, melanoma, lympho-
ma, orbital, and adnexal tumors in ophthalmic pathology will have the best chance for consid-
eration in this theme issue. Manuscripts must be submitted no later than February 1, 2009 for
consideration in the joint theme issue. All submissions will undergo our usual peer review process.
Important: When submitting a manuscript for this theme issue, be certain to mention this in
both the cover letter and the Comment section within the AllenTrack submission system.
To view our Instructions for Authors, visit
http://arpa.allenpress.com/pdf/instructions㛮for㛮authors.pdf

More Related Content

Similar to Gastritis.pdf

H. pylori past, present and future
H. pylori past, present and futureH. pylori past, present and future
H. pylori past, present and futureSameh Badr
 
Bacterium helicobacter pylori
Bacterium helicobacter pyloriBacterium helicobacter pylori
Bacterium helicobacter pyloriTAYYAB MUNEER
 
Bohomolets Surgery 4th year Lecture #7
Bohomolets Surgery 4th year Lecture #7Bohomolets Surgery 4th year Lecture #7
Bohomolets Surgery 4th year Lecture #7Dr. Rubz
 
H Pylori Management 2023 .pptx
H Pylori Management 2023 .pptxH Pylori Management 2023 .pptx
H Pylori Management 2023 .pptxDrChernHaoChong
 
FOCUS ON CURRENT TRENDS IN THE TREATMENT OF HELICOBACTER PYLORI INFECTION
FOCUS ON CURRENT TRENDS IN THE TREATMENT OF HELICOBACTER PYLORI INFECTIONFOCUS ON CURRENT TRENDS IN THE TREATMENT OF HELICOBACTER PYLORI INFECTION
FOCUS ON CURRENT TRENDS IN THE TREATMENT OF HELICOBACTER PYLORI INFECTIONSarvan Mani
 
Disseminated histoplasmosis intestinal multiple ulcers without gastrointestin...
Disseminated histoplasmosis intestinal multiple ulcers without gastrointestin...Disseminated histoplasmosis intestinal multiple ulcers without gastrointestin...
Disseminated histoplasmosis intestinal multiple ulcers without gastrointestin...Mario Fernando Dueñas Patólogo.
 
Stomach pathology lecture
Stomach pathology lectureStomach pathology lecture
Stomach pathology lectureDrsapna Harsha
 
L2. Diarrhoea disease and Food poisoning.pptx
L2. Diarrhoea disease and Food poisoning.pptxL2. Diarrhoea disease and Food poisoning.pptx
L2. Diarrhoea disease and Food poisoning.pptxdanielmwandu
 
H.Pylori and peptic ulcer.pptx
H.Pylori and peptic ulcer.pptxH.Pylori and peptic ulcer.pptx
H.Pylori and peptic ulcer.pptxPradeep Pande
 
Topic: Gastritis Nursing Lecture ppt.ppt
Topic: Gastritis Nursing Lecture ppt.pptTopic: Gastritis Nursing Lecture ppt.ppt
Topic: Gastritis Nursing Lecture ppt.pptShashi Prakash
 
Non neoplastic disorders of esophagus
Non neoplastic disorders of esophagusNon neoplastic disorders of esophagus
Non neoplastic disorders of esophagusVas Kannan
 
Lect 4- gastric disorder
Lect 4- gastric disorderLect 4- gastric disorder
Lect 4- gastric disorderMohanad Mohanad
 
Gastritis.pptx Gastro Intestinal Disorder
Gastritis.pptx Gastro Intestinal DisorderGastritis.pptx Gastro Intestinal Disorder
Gastritis.pptx Gastro Intestinal DisorderPatelVedanti
 

Similar to Gastritis.pdf (20)

Peptic ulcer modified 2
Peptic ulcer modified 2Peptic ulcer modified 2
Peptic ulcer modified 2
 
peptic ulcer
peptic ulcerpeptic ulcer
peptic ulcer
 
H. pylori past, present and future
H. pylori past, present and futureH. pylori past, present and future
H. pylori past, present and future
 
Bacterium helicobacter pylori
Bacterium helicobacter pyloriBacterium helicobacter pylori
Bacterium helicobacter pylori
 
Bohomolets Surgery 4th year Lecture #7
Bohomolets Surgery 4th year Lecture #7Bohomolets Surgery 4th year Lecture #7
Bohomolets Surgery 4th year Lecture #7
 
Lect 4- gastric disorder
Lect 4- gastric disorderLect 4- gastric disorder
Lect 4- gastric disorder
 
H Pylori Management 2023 .pptx
H Pylori Management 2023 .pptxH Pylori Management 2023 .pptx
H Pylori Management 2023 .pptx
 
Helicobacter
HelicobacterHelicobacter
Helicobacter
 
FOCUS ON CURRENT TRENDS IN THE TREATMENT OF HELICOBACTER PYLORI INFECTION
FOCUS ON CURRENT TRENDS IN THE TREATMENT OF HELICOBACTER PYLORI INFECTIONFOCUS ON CURRENT TRENDS IN THE TREATMENT OF HELICOBACTER PYLORI INFECTION
FOCUS ON CURRENT TRENDS IN THE TREATMENT OF HELICOBACTER PYLORI INFECTION
 
Disseminated histoplasmosis intestinal multiple ulcers without gastrointestin...
Disseminated histoplasmosis intestinal multiple ulcers without gastrointestin...Disseminated histoplasmosis intestinal multiple ulcers without gastrointestin...
Disseminated histoplasmosis intestinal multiple ulcers without gastrointestin...
 
Stomach pathology lecture
Stomach pathology lectureStomach pathology lecture
Stomach pathology lecture
 
L2. Diarrhoea disease and Food poisoning.pptx
L2. Diarrhoea disease and Food poisoning.pptxL2. Diarrhoea disease and Food poisoning.pptx
L2. Diarrhoea disease and Food poisoning.pptx
 
H.Pylori and peptic ulcer.pptx
H.Pylori and peptic ulcer.pptxH.Pylori and peptic ulcer.pptx
H.Pylori and peptic ulcer.pptx
 
Topic: Gastritis Nursing Lecture ppt.ppt
Topic: Gastritis Nursing Lecture ppt.pptTopic: Gastritis Nursing Lecture ppt.ppt
Topic: Gastritis Nursing Lecture ppt.ppt
 
Dyspepsia
DyspepsiaDyspepsia
Dyspepsia
 
12.Peritonitis.pdf
12.Peritonitis.pdf12.Peritonitis.pdf
12.Peritonitis.pdf
 
Non neoplastic disorders of esophagus
Non neoplastic disorders of esophagusNon neoplastic disorders of esophagus
Non neoplastic disorders of esophagus
 
Lect 4- gastric disorder
Lect 4- gastric disorderLect 4- gastric disorder
Lect 4- gastric disorder
 
Lect 4- gastric disord
Lect 4- gastric disordLect 4- gastric disord
Lect 4- gastric disord
 
Gastritis.pptx Gastro Intestinal Disorder
Gastritis.pptx Gastro Intestinal DisorderGastritis.pptx Gastro Intestinal Disorder
Gastritis.pptx Gastro Intestinal Disorder
 

Recently uploaded

Call Girls Service Chennai Jiya 7001305949 Independent Escort Service Chennai
Call Girls Service Chennai Jiya 7001305949 Independent Escort Service ChennaiCall Girls Service Chennai Jiya 7001305949 Independent Escort Service Chennai
Call Girls Service Chennai Jiya 7001305949 Independent Escort Service ChennaiNehru place Escorts
 
Call Girls Horamavu WhatsApp Number 7001035870 Meeting With Bangalore Escorts
Call Girls Horamavu WhatsApp Number 7001035870 Meeting With Bangalore EscortsCall Girls Horamavu WhatsApp Number 7001035870 Meeting With Bangalore Escorts
Call Girls Horamavu WhatsApp Number 7001035870 Meeting With Bangalore Escortsvidya singh
 
Call Girls Yelahanka Just Call 7001305949 Top Class Call Girl Service Available
Call Girls Yelahanka Just Call 7001305949 Top Class Call Girl Service AvailableCall Girls Yelahanka Just Call 7001305949 Top Class Call Girl Service Available
Call Girls Yelahanka Just Call 7001305949 Top Class Call Girl Service Availablenarwatsonia7
 
Call Girls Service Noida Maya 9711199012 Independent Escort Service Noida
Call Girls Service Noida Maya 9711199012 Independent Escort Service NoidaCall Girls Service Noida Maya 9711199012 Independent Escort Service Noida
Call Girls Service Noida Maya 9711199012 Independent Escort Service NoidaPooja Gupta
 
Bangalore Call Girls Majestic 📞 9907093804 High Profile Service 100% Safe
Bangalore Call Girls Majestic 📞 9907093804 High Profile Service 100% SafeBangalore Call Girls Majestic 📞 9907093804 High Profile Service 100% Safe
Bangalore Call Girls Majestic 📞 9907093804 High Profile Service 100% Safenarwatsonia7
 
Kesar Bagh Call Girl Price 9548273370 , Lucknow Call Girls Service
Kesar Bagh Call Girl Price 9548273370 , Lucknow Call Girls ServiceKesar Bagh Call Girl Price 9548273370 , Lucknow Call Girls Service
Kesar Bagh Call Girl Price 9548273370 , Lucknow Call Girls Servicemakika9823
 
Russian Call Girls in Chennai Pallavi 9907093804 Independent Call Girls Servi...
Russian Call Girls in Chennai Pallavi 9907093804 Independent Call Girls Servi...Russian Call Girls in Chennai Pallavi 9907093804 Independent Call Girls Servi...
Russian Call Girls in Chennai Pallavi 9907093804 Independent Call Girls Servi...Nehru place Escorts
 
Call Girls Service Bellary Road Just Call 7001305949 Enjoy College Girls Service
Call Girls Service Bellary Road Just Call 7001305949 Enjoy College Girls ServiceCall Girls Service Bellary Road Just Call 7001305949 Enjoy College Girls Service
Call Girls Service Bellary Road Just Call 7001305949 Enjoy College Girls Servicenarwatsonia7
 
Call Girls Doddaballapur Road Just Call 7001305949 Top Class Call Girl Servic...
Call Girls Doddaballapur Road Just Call 7001305949 Top Class Call Girl Servic...Call Girls Doddaballapur Road Just Call 7001305949 Top Class Call Girl Servic...
Call Girls Doddaballapur Road Just Call 7001305949 Top Class Call Girl Servic...narwatsonia7
 
Russian Call Girls Chennai Madhuri 9907093804 Independent Call Girls Service ...
Russian Call Girls Chennai Madhuri 9907093804 Independent Call Girls Service ...Russian Call Girls Chennai Madhuri 9907093804 Independent Call Girls Service ...
Russian Call Girls Chennai Madhuri 9907093804 Independent Call Girls Service ...Nehru place Escorts
 
Artifacts in Nuclear Medicine with Identifying and resolving artifacts.
Artifacts in Nuclear Medicine with Identifying and resolving artifacts.Artifacts in Nuclear Medicine with Identifying and resolving artifacts.
Artifacts in Nuclear Medicine with Identifying and resolving artifacts.MiadAlsulami
 
Call Girl Chennai Indira 9907093804 Independent Call Girls Service Chennai
Call Girl Chennai Indira 9907093804 Independent Call Girls Service ChennaiCall Girl Chennai Indira 9907093804 Independent Call Girls Service Chennai
Call Girl Chennai Indira 9907093804 Independent Call Girls Service ChennaiNehru place Escorts
 
Call Girl Service Bidadi - For 7001305949 Cheap & Best with original Photos
Call Girl Service Bidadi - For 7001305949 Cheap & Best with original PhotosCall Girl Service Bidadi - For 7001305949 Cheap & Best with original Photos
Call Girl Service Bidadi - For 7001305949 Cheap & Best with original Photosnarwatsonia7
 
Call Girls Service Pune Vaishnavi 9907093804 Short 1500 Night 6000 Best call ...
Call Girls Service Pune Vaishnavi 9907093804 Short 1500 Night 6000 Best call ...Call Girls Service Pune Vaishnavi 9907093804 Short 1500 Night 6000 Best call ...
Call Girls Service Pune Vaishnavi 9907093804 Short 1500 Night 6000 Best call ...Miss joya
 
Call Girls In Andheri East Call 9920874524 Book Hot And Sexy Girls
Call Girls In Andheri East Call 9920874524 Book Hot And Sexy GirlsCall Girls In Andheri East Call 9920874524 Book Hot And Sexy Girls
Call Girls In Andheri East Call 9920874524 Book Hot And Sexy Girlsnehamumbai
 
Russian Call Girls in Pune Tanvi 9907093804 Short 1500 Night 6000 Best call g...
Russian Call Girls in Pune Tanvi 9907093804 Short 1500 Night 6000 Best call g...Russian Call Girls in Pune Tanvi 9907093804 Short 1500 Night 6000 Best call g...
Russian Call Girls in Pune Tanvi 9907093804 Short 1500 Night 6000 Best call g...Miss joya
 
Low Rate Call Girls Ambattur Anika 8250192130 Independent Escort Service Amba...
Low Rate Call Girls Ambattur Anika 8250192130 Independent Escort Service Amba...Low Rate Call Girls Ambattur Anika 8250192130 Independent Escort Service Amba...
Low Rate Call Girls Ambattur Anika 8250192130 Independent Escort Service Amba...narwatsonia7
 
Russian Call Girls Chickpet - 7001305949 Booking and charges genuine rate for...
Russian Call Girls Chickpet - 7001305949 Booking and charges genuine rate for...Russian Call Girls Chickpet - 7001305949 Booking and charges genuine rate for...
Russian Call Girls Chickpet - 7001305949 Booking and charges genuine rate for...narwatsonia7
 
Bangalore Call Girls Marathahalli 📞 9907093804 High Profile Service 100% Safe
Bangalore Call Girls Marathahalli 📞 9907093804 High Profile Service 100% SafeBangalore Call Girls Marathahalli 📞 9907093804 High Profile Service 100% Safe
Bangalore Call Girls Marathahalli 📞 9907093804 High Profile Service 100% Safenarwatsonia7
 

Recently uploaded (20)

Call Girls Service Chennai Jiya 7001305949 Independent Escort Service Chennai
Call Girls Service Chennai Jiya 7001305949 Independent Escort Service ChennaiCall Girls Service Chennai Jiya 7001305949 Independent Escort Service Chennai
Call Girls Service Chennai Jiya 7001305949 Independent Escort Service Chennai
 
Call Girls Horamavu WhatsApp Number 7001035870 Meeting With Bangalore Escorts
Call Girls Horamavu WhatsApp Number 7001035870 Meeting With Bangalore EscortsCall Girls Horamavu WhatsApp Number 7001035870 Meeting With Bangalore Escorts
Call Girls Horamavu WhatsApp Number 7001035870 Meeting With Bangalore Escorts
 
Call Girls Yelahanka Just Call 7001305949 Top Class Call Girl Service Available
Call Girls Yelahanka Just Call 7001305949 Top Class Call Girl Service AvailableCall Girls Yelahanka Just Call 7001305949 Top Class Call Girl Service Available
Call Girls Yelahanka Just Call 7001305949 Top Class Call Girl Service Available
 
Call Girls Service Noida Maya 9711199012 Independent Escort Service Noida
Call Girls Service Noida Maya 9711199012 Independent Escort Service NoidaCall Girls Service Noida Maya 9711199012 Independent Escort Service Noida
Call Girls Service Noida Maya 9711199012 Independent Escort Service Noida
 
Bangalore Call Girls Majestic 📞 9907093804 High Profile Service 100% Safe
Bangalore Call Girls Majestic 📞 9907093804 High Profile Service 100% SafeBangalore Call Girls Majestic 📞 9907093804 High Profile Service 100% Safe
Bangalore Call Girls Majestic 📞 9907093804 High Profile Service 100% Safe
 
Kesar Bagh Call Girl Price 9548273370 , Lucknow Call Girls Service
Kesar Bagh Call Girl Price 9548273370 , Lucknow Call Girls ServiceKesar Bagh Call Girl Price 9548273370 , Lucknow Call Girls Service
Kesar Bagh Call Girl Price 9548273370 , Lucknow Call Girls Service
 
Russian Call Girls in Chennai Pallavi 9907093804 Independent Call Girls Servi...
Russian Call Girls in Chennai Pallavi 9907093804 Independent Call Girls Servi...Russian Call Girls in Chennai Pallavi 9907093804 Independent Call Girls Servi...
Russian Call Girls in Chennai Pallavi 9907093804 Independent Call Girls Servi...
 
Call Girls Service Bellary Road Just Call 7001305949 Enjoy College Girls Service
Call Girls Service Bellary Road Just Call 7001305949 Enjoy College Girls ServiceCall Girls Service Bellary Road Just Call 7001305949 Enjoy College Girls Service
Call Girls Service Bellary Road Just Call 7001305949 Enjoy College Girls Service
 
Call Girls Doddaballapur Road Just Call 7001305949 Top Class Call Girl Servic...
Call Girls Doddaballapur Road Just Call 7001305949 Top Class Call Girl Servic...Call Girls Doddaballapur Road Just Call 7001305949 Top Class Call Girl Servic...
Call Girls Doddaballapur Road Just Call 7001305949 Top Class Call Girl Servic...
 
Russian Call Girls Chennai Madhuri 9907093804 Independent Call Girls Service ...
Russian Call Girls Chennai Madhuri 9907093804 Independent Call Girls Service ...Russian Call Girls Chennai Madhuri 9907093804 Independent Call Girls Service ...
Russian Call Girls Chennai Madhuri 9907093804 Independent Call Girls Service ...
 
Artifacts in Nuclear Medicine with Identifying and resolving artifacts.
Artifacts in Nuclear Medicine with Identifying and resolving artifacts.Artifacts in Nuclear Medicine with Identifying and resolving artifacts.
Artifacts in Nuclear Medicine with Identifying and resolving artifacts.
 
Call Girl Chennai Indira 9907093804 Independent Call Girls Service Chennai
Call Girl Chennai Indira 9907093804 Independent Call Girls Service ChennaiCall Girl Chennai Indira 9907093804 Independent Call Girls Service Chennai
Call Girl Chennai Indira 9907093804 Independent Call Girls Service Chennai
 
Call Girl Service Bidadi - For 7001305949 Cheap & Best with original Photos
Call Girl Service Bidadi - For 7001305949 Cheap & Best with original PhotosCall Girl Service Bidadi - For 7001305949 Cheap & Best with original Photos
Call Girl Service Bidadi - For 7001305949 Cheap & Best with original Photos
 
Call Girls Service Pune Vaishnavi 9907093804 Short 1500 Night 6000 Best call ...
Call Girls Service Pune Vaishnavi 9907093804 Short 1500 Night 6000 Best call ...Call Girls Service Pune Vaishnavi 9907093804 Short 1500 Night 6000 Best call ...
Call Girls Service Pune Vaishnavi 9907093804 Short 1500 Night 6000 Best call ...
 
Call Girls In Andheri East Call 9920874524 Book Hot And Sexy Girls
Call Girls In Andheri East Call 9920874524 Book Hot And Sexy GirlsCall Girls In Andheri East Call 9920874524 Book Hot And Sexy Girls
Call Girls In Andheri East Call 9920874524 Book Hot And Sexy Girls
 
Russian Call Girls in Pune Tanvi 9907093804 Short 1500 Night 6000 Best call g...
Russian Call Girls in Pune Tanvi 9907093804 Short 1500 Night 6000 Best call g...Russian Call Girls in Pune Tanvi 9907093804 Short 1500 Night 6000 Best call g...
Russian Call Girls in Pune Tanvi 9907093804 Short 1500 Night 6000 Best call g...
 
Russian Call Girls in Delhi Tanvi ➡️ 9711199012 💋📞 Independent Escort Service...
Russian Call Girls in Delhi Tanvi ➡️ 9711199012 💋📞 Independent Escort Service...Russian Call Girls in Delhi Tanvi ➡️ 9711199012 💋📞 Independent Escort Service...
Russian Call Girls in Delhi Tanvi ➡️ 9711199012 💋📞 Independent Escort Service...
 
Low Rate Call Girls Ambattur Anika 8250192130 Independent Escort Service Amba...
Low Rate Call Girls Ambattur Anika 8250192130 Independent Escort Service Amba...Low Rate Call Girls Ambattur Anika 8250192130 Independent Escort Service Amba...
Low Rate Call Girls Ambattur Anika 8250192130 Independent Escort Service Amba...
 
Russian Call Girls Chickpet - 7001305949 Booking and charges genuine rate for...
Russian Call Girls Chickpet - 7001305949 Booking and charges genuine rate for...Russian Call Girls Chickpet - 7001305949 Booking and charges genuine rate for...
Russian Call Girls Chickpet - 7001305949 Booking and charges genuine rate for...
 
Bangalore Call Girls Marathahalli 📞 9907093804 High Profile Service 100% Safe
Bangalore Call Girls Marathahalli 📞 9907093804 High Profile Service 100% SafeBangalore Call Girls Marathahalli 📞 9907093804 High Profile Service 100% Safe
Bangalore Call Girls Marathahalli 📞 9907093804 High Profile Service 100% Safe
 

Gastritis.pdf

  • 1. 1586 Arch Pathol Lab Med—Vol 132, October 2008 Pathologic Diagnosis of Gastritis—Sepulveda & Patil Practical Approach to the Pathologic Diagnosis of Gastritis Antonia R. Sepulveda, MD, PhD; Madhavi Patil, MD ● Context.—Most types of gastritis can be diagnosed on hematoxylin-eosin stains. The most common type of chron- ic gastritis is Helicobacter pylori gastritis. Reactive or chemical gastropathy, which is often associated with non- steroidal anti-inflammatory drug use or bile reflux, is com- mon in most practices. The diagnosis of atrophic gastritis can be challenging if few biopsy samples are available and if the location of the biopsies in the stomach is not known, such as when random biopsies are sampled in one jar. If the biopsy site is not known, immunohistochemical stains, such as a combination of synaptophysin and gastrin, are useful in establishing the biopsy location. Objective.—To demonstrate a practical approach to achieving a pathologic diagnosis of gastritis by evaluating a limited number of features in mucosal biopsies. Data Source.—In this article, we present several repre- sentative gastric biopsy cases from a gastrointestinal pa- thology practice to demonstrate the practical application of basic histopathologic methods for the diagnosis of gas- tritis. Conclusions.—Limited ancillary tests are usually re- quired for a diagnosis of gastritis. In some cases, special stains, such as acid-fast stains, and immunohistochemical stains, such as for H pylori and viruses, can be useful. Hel- icobacter pylori immunohistochemical stains can particu- larly contribute (1) when moderate to severe, chronic gas- tritis or active gastritis is present but no Helicobacter or- ganisms are identified upon hematoxylin-eosin stain; (2) when extensive intestinal metaplasia is present; and (3) in follow-up biopsies, after antibiotic treatment for H pylori. (Arch Pathol Lab Med. 2008;132:1586–1593) Gastritis refers to a group of diseases characterized by inflammation of the gastric mucosa. Histologic ex- amination of gastric mucosal biopsies is necessary to es- tablish a diagnosis of gastritis. In clinical practice, the role of the pathologist who evaluates a gastric biopsy for gas- tritis is to find the cause of gastritis because that will pro- vide direct targets toward which therapeutic measures can be directed. An etiologic classification of gastritis is pre- sented at the end of this section. Comprehensive reviews of gastritis have been published.1,2,3 The goal of this article is to present a practical approach to the diagnosis of the most common types of gastritis encountered in a large practice of gastrointestinal pathology. The reader will be presented several cases representative of typical forms of gastritis; for each case, the reader will be prompted through a series of questions to examine the histologic features of the mucosa, leading to a pattern of answers and to a final diagnosis. The first question is aimed at determining whether or not there are features of chronic or acute (active) gastritis Accepted for publication January 10, 2008. From the Department of Pathology and Laboratory Medicine, Hos- pital of the University of Pennsylvania, Philadelphia. The authors have no relevant financial interest in the products or companies described in this article. Presented in part at the 47th Annual Meeting of the Houston Society of Clinical Pathologists, Houston, Tex, April 21, 2007. Reprints: Antonia R. Sepulveda, MD, PhD, Department of Pathology and Laboratory Medicine, University of Pennsylvania, 3400 Spruce St, Founders Six, Philadelphia, PA 19104 (e-mail: asepu@mail.med. upenn.edu). present. If the biopsy shows chronic gastritis, the follow- ing questions should be posed: 1. Are there features of chronic gastritis present? Lym- phocytic and plasmacytic inflammatory reaction indicates chronic gastritis. 2. Are there neutrophils in the mucosa? The presence of neutrophils indicate active gastritis. 3. Is there Helicobacter? 4. Is there glandular atrophy? Is intestinal metaplasia present? 5. What is the topography of lesions (predominantly in the oxyntic mucosa of the body and fundus, predomi- nantly in antrum, or involving both locations)? 6. Are there special features (such as granulomas, fo- veolar hyperplasia, viral inclusions)? 7. What ancillary studies are indicated, and what are the results? TYPES OF CHRONIC GASTRITIS Infectious Gastritis Helicobacter pylori infection is the most common cause of chronic gastritis. Other forms of infectious gastritis include the following: Helicobacter heilmannii–associated gastritis; granulomatous gastritis associated with gastric infections in mycobacteriosis, syphilis, histoplasmosis, mucormyco- sis, South American blastomycosis, anisakiasis or anisak- idosis; chronic gastritis associated with parasitic infec- tions; and viral infections, such as cytomegalovirus and herpesvirus infection.
  • 2. Arch Pathol Lab Med—Vol 132, October 2008 Pathologic Diagnosis of Gastritis—Sepulveda & Patil 1587 Figure 1. Helicobacter pylori–associated chronic active gastritis. A, Chronic inflammation oriented toward the surface of the mucosa. Neu- trophils cannot be seen at this magnification (original magnification ⫻10) but were identified with high power (hematoxylin-eosin stain). B, Circled area shows H pylori organisms within the mucus layer close to the surface of gastric epithelial cells (hematoxylin-eosin, original magnification ⫻40). C, Gastric mucosa with intestinal metaplasia (he- matoxylin-eosin, original magnification ⫻20). Noninfectious Gastritis Noninfectious gastritis is associated with autoimmune gastritis; reactive or chemical gastropathy, usually related to chronic bile reflux or nonsteroidal anti-inflammatory drug (NSAID) intake; uremic gastropathy; noninfectious granulomatous gastritis; lymphocytic gastritis, including gastritis associated with celiac disease; eosinophilic gas- tritis; radiation injury to the stomach; graft-versus-host disease; ischemic gastritis; and gastritis secondary to che- motherapy. Many cases of gastritis are of undetermined cause and present as chronic, inactive gastritis with various degrees of severity.3 TYPES OF ACUTE GASTRITIS Many of the forms of chronic gastritis may present with an acute form, with progression to chronic gastritis be- cause of persisting injury or sequelae. This is the case of gastritis associated with long-term intake of aspirin and other NSAIDs and bile reflux into the stomach; excessive alcohol consumption; heavy smoking; cancer chemother- apeutic drugs and radiation; acids and alkali in suicide attempts; uremia; severe stress (trauma, burns, surgery); ischemia and shock; systemic infections; mechanical trau- ma, such as intubation associated mucosal lesions; and vi- ral infections. Case 1 A 60-year-old man underwent esophagogastroduoden- oscopy. A biopsy of gastric antrum was submitted to pa- thology to rule out H pylori. The histologic findings are shown in Figure 1, A through C. Findings. Examination of the biopsy material available gives the following answers: 1. Are there features of chronic gastritis? Yes. The gas- tric antral mucosa shows expansion of the lamina propria by chronic inflammatory cells, consisting of plasma cells and small lymphocytes, predominantly located toward the luminal aspect of the mucosa, a pattern that is suggestive of H pylori infection. 2. Are there neutrophils in the mucosa? Yes. Therefore, this represents active gastritis. This is a mild form of active gastritis. 3. Is there Helicobacter? Yes. Hematoxylin-eosin (H&E) examination reveals diagnostic H pylori bacterial forms in the surface mucus layer in close proximity to the apical aspect of surface epithelial cells. 4. Is there glandular atrophy? The biopsy sample avail- able is not adequate for evaluation of atrophic gastritis; multiple biopsies, including samples of gastric body, are necessary for adequate evaluation of glandular atrophy. Is there intestinal metaplasia? Yes. 5. What is the topography of lesions? The chronic gas- tritis in this case involves, at minimum, the gastric antrum; it is advisable to obtain biopsy samples of both gastric antrum and body for a better evaluation of gastritis, as recommended by the updated Sydney guidelines4 for clas- sification of gastritis. 6. Are additional special features present? No. 7. Are special stains recommended? No. Diagnosis. Gastric antral mucosa with H pylori–asso- ciated chronic gastritis, mildly active, and focal intestinal metaplasia. H PYLORI–ASSOCIATED CHRONIC GASTRITIS The Helicobacter species consist of gram-negative rods that infect the gastric mucosa. Helicobacter pylori bacteria are 3.5 ␮m long and are generally comma-shaped or have slightly spiral forms. Helicobacter heilmannii, a rare agent of chronic gastritis, is a 5- to 9-␮m-long bacterium, with a characteristic tightly corkscrew-shaped, spiral form.5 Helicobacter pylori infection usually is acquired during childhood, persisting as chronic gastritis if the organism is not eradicated. During progression of gastritis over the years, the gastric mucosa undergoes a sequence of changes that may lead to glandular atrophy, intestinal metaplasia, increased risk of gastric dysplasia and carcinoma,6–9 and mucosa-associated lymphoid tissue lymphoma,10,11 report- ed as extranodal, marginal zone, B-cell lymphoma in the World Health Organization classification.12 Helicobacter pylori infection is associated with the histo- logic pattern of active and chronic gastritis, reflecting the presence of neutrophils and mononuclear cells (lympho- cytes and plasma cells) in the mucosa, respectively. The term active gastritis is preferred to acute gastritis because H pylori gastritis is a long-standing chronic infection with ongoing activity. Lymphoid aggregates and lymphoid fol- licles may be observed expanding the lamina propria, and rare lymphocytes may enter the epithelium. Helicobacter pylori organisms are found within the gastric mucus layer that overlays the apical side of gastric surface cells, and lower numbers are found in the lower portions of the gas- tric foveolae. Helicobacter pylori may be found within the deeper areas of the mucosa in association with glandular cells in patients on acid blockers, such as the commonly used proton pump inhibitors.13 Helicobacter pylori–associated gastritis can display differ- ent levels of severity. The severity of H pylori gastritis ac- tivity may be indicated in a pathology report as mild (rare neutrophils seen), moderate (obvious neutrophils within the glandular and foveolar epithelium), or severe (numer- ous neutrophils with glandular microabscesses and mu- cosal erosion or frank ulceration).4,14 Helicobacter pylori–associated chronic gastritis can mani- fest as a pangastritis involving the area from the pylorus to the gastric body and cardia, or it may predominantly involve the antrum. Patients with gastric ulcers generally have antral-predominant gastritis, whereas pangastritis,
  • 3. 1588 Arch Pathol Lab Med—Vol 132, October 2008 Pathologic Diagnosis of Gastritis—Sepulveda & Patil Figure 2. Chronic active gastritis in a patient with Crohn disease. A glandular abscess is shown; additionally, there are many neutrophils in the lamina propria admixed with a background of chronic inflamma- tion (hematoxylin-eosin, original magnification ⫻20). or at least multifocal gastritis, is more common in patients with gastric carcinoma. The latter generally have signifi- cant intestinal metaplasia and gastric oxyntic glandular atrophy coexisting in the background stomach. It is im- portant to make a pathologic diagnosis of atrophic gastri- tis because gastric atrophy is associated with increased risk of gastric cancer.15,16 Patients with chronic atrophic gastritis may have up to a 16-fold increased risk of devel- oping gastric carcinoma, compared with the general pop- ulation.15,17 When large numbers of H pylori are present in the mu- cosa, the identification of typical organisms is generally possible on H&E stains. However, there are cases of chron- ic, active gastritis with features suggestive of H pylori gas- tritis in which the organisms are not detected. Several spe- cial stains have been extensively used to help identify H pylori organisms in the gastric mucosa, including modi- fied-Giemsa, Genta, thiazine stains, and immunohisto- chemistry against Helicobacter antigens. The selection of the special stain used is largely dependent on preferences re- lated to individual practices. Although, overall, no major differences in sensitivity and specificity have been report- ed, studies have recommended immunohistochemical stains in a subset of cases.18,19 In our practice, we prefer to use immunohistochemical stains for detection of H pylori if organisms are not found on H&E stains in the following cases: (1) if moderate to severe chronic gastritis or any grade of active gastritis is present but no Helicobacter or- ganisms are identified on H&E; (2) when extensive intes- tinal metaplasia is present because H pylori density is re- duced in areas of intestinal metaplasia; and (3) during follow-up biopsies after antibiotic treatment for H pylori. Helicobacter heilmannii may cause similar pathology, and the treatment is similar to H pylori.5 Case 2 A 45-year-old man is seen to rule out H pylori. He pre- sents with a history of Crohn disease. The histologic find- ings are shown in Figure 2. Findings. Examination of the biopsy material results in the following pattern of answers: 1. Are there features of chronic gastritis? Yes. The gas- tric antral mucosa shows expansion of the lamina propria by chronic inflammatory cells, consisting of admixed plas- ma cells and small lymphocytes, throughout the thickness of the mucosa. 2. Are there neutrophils in the mucosa? Yes, with an occasional glandular abscess; therefore, there is active gas- tritis. Of note, the active gastritis has a patchy distribution. 3. Is there Helicobacter? No. Examination with H&E stain does not reveal such bacterial forms. Immunohisto- chemical stain is performed. 4. Is there atrophy? The biopsy sample available is not adequate for evaluation of atrophic gastritis because the biopsy material is only from the gastric antrum; multiple gastric body biopsies are necessary for adequate evalua- tion of glandular atrophy. There is no intestinal metapla- sia. 5. What is the topography of lesions? The chronic gas- tritis involves, at minimum, the gastric antrum. 6. Are additional special features seen? No. Although in a case of Crohn disease gastritis, epithelioid granulo- mas may be present; in this case, no granulomas were seen. 7. Are special stains recommended? Yes. Helicobacter py- lori immunohistochemical stain, which is helpful in cases where Crohn disease is suspected because the absence of H pylori organisms in chronic active gastritis is consistent with Crohn disease. The H pylori immunohistochemical stain in this case is negative. Diagnosis. Gastric antral mucosa with chronic active gastritis, moderately active, patchy. No H pylori organisms are identified by H&E or immunohistochemistry. Note: These features are consistent with Crohn disease–associ- ated gastritis. CROHN DISEASE–ASSOCIATED GASTRITIS The hallmark histopathologic features of Crohn disease– associated gastritis are the presence of patchy, acute in- flammation with possible gastric pit or glandular abscess- es, commonly with a background with lymphoid aggre- gates. Recent studies20 reported the presentation of gastri- tis in patients with Crohn disease as a focally enhanced gastritis, characterized by small collections of lymphocytes and histiocytes surrounding a small group of gastric fo- veolae or glands, often with infiltrates of neutrophils. In severe cases, there may be diffuse inflammation in the lamina propria, with variable glandular loss, fissures, ul- cers, transmural inflammation, and fibrosis. Noncaseating epithelioid granulomas may be present in about one third of cases of Crohn disease gastritis but are often not seen, at least in part, because of limited tissue sampling. When granulomas are identified, the differential diag- nosis includes other forms of granulomatous gastritis. There are infectious and noninfectious causes of granulo- matous gastritis. Noninfectious diseases represent the usu- al cause of gastric granulomas and include Crohn disease, sarcoidosis, and isolated granulomatous gastritis. Sarcoid- like granulomas may be observed in cocaine users, and foreign material is occasionally observed in the granulo- mas. Sarcoidosis of the stomach is usually associated with granulomas in other organs, especially the lungs, hilar nodes, or salivary glands. A diagnosis of idiopathic, iso-
  • 4. Arch Pathol Lab Med—Vol 132, October 2008 Pathologic Diagnosis of Gastritis—Sepulveda & Patil 1589 Figure 3. Helicobacter pylori–associated atrophic gastritis. A, Chronic active gastritis involving the gastric oxyntic mucosa with glandular at- rophy (hematoxylin-eosin, original magnification ⫻10). Inset shows rare neutrophils into the glandular epithelium (white arrows). B, Im- munohistochemical stain for H pylori shows a small area with organ- isms attached to the surface epithelium. Insets show individual Heli- cobacter bacteria (thin arrows) with characteristic elongated, slightly spiral S shape or clusters of packed bacteria (thick arrowhead) closely adherent to the surface of epithelial cells. Immunohistochemical stains are useful in cases such as this one, when H pylori organisms are closely associated with the surface epithelial cells making it difficult to ascertain the characteristic bacterial morphology on hematoxylin-eosin stains (original magnification ⫻40). lated, granulomatous gastritis is rendered when known entities associated with granulomas are excluded. Case 3 A 60-year-old man presents with a nodularity of the gastric body to rule out H pylori. Esophagogastroduoden- oscopy with biopsy of the nodular areas was performed. The histologic findings are shown in Figure 3, A and B. Findings. Examination of the biopsy material results in the following pattern of answers: 1. Are there features of chronic gastritis? Yes. 2. Are there neutrophils in the mucosa? Yes. There are neutrophils in the mucosa, representing active gastritis. 3. Is there Helicobacter? No. Examination with H&E stain does not reveal H pylori bacterial forms. Immunohis- tochemical is performed. 4. Is there atrophy? Yes. There is a reduced number of oxyntic glands in the biopsy. There is no intestinal meta- plasia. 5. What is the topography of lesions? The chronic gas- tritis involves, at minimum, the gastric body. 6. Are additional special features seen? No. 7. Are special stains recommended? Yes. Helicobacter py- lori immunohistochemical stain, which is positive. Diagnosis. Gastric oxyntic mucosa with H pylori–as- sociated chronic active gastritis and glandular atrophy, moderate. No intestinal metaplasia is identified. Helicobac- ter organisms are identified by immunohistochemistry. ATROPHIC GASTRITIS Several publications, including those reporting the Syd- ney system and the updated Houston classification of gas- tritis, have proposed criteria for the evaluation of atrophic gastritis. Interobserver variability is significant, especially in the evaluation of antral atrophy.4,21 Recent advances that appear to decrease the interobserver variation in the as- sessment of gastric atrophy have been reported.14 Atrophy is more accurately assessed after resolution of severe in- flammation of the mucosa; therefore, if there is H pylori gastritis, the infection should be eradicated before atrophy is difinitively evaluated. When marked inflammation is present, a diagnosis of indefinite for atrophy may be of- fered, especially if there is no intestinal metaplasia. The recommended definition of atrophy is the loss of appropriate glands, and atrophy can be scored according to the degree of severity as mild, moderate, or severe.22 In this definition, intestinal metaplasia represents a form of atrophy described as metaplastic atrophy (or gastric glan- dular atrophy with intestinal metaplasia). Gastric atrophy is usually associated with intestinal metaplasia. However, in limited endoscopic biopsies, in- testinal metaplasia might not be sampled, whereas the mucosa shows definitive atrophy. Usually gastric atrophy and intestinal metaplasia occur on a background of chron- ic gastritis, hence the term atrophic gastritis. Sampling of the mucosa for evaluation of atrophy and gastritis is generally adequate by using the 5 biopsies rec- ommended by the Sydney system, including 2 biopsies from the antrum, 2 from the corpus or body, and 1 from the incisura angularis.4,21 It is essential for the pathologist to have a means of determining the specific site in the stomach where a biopsy is sampled from because specific topography of atrophy characterizes the different types of atrophic gastritis. In atrophic gastritis associated with H pylori, glandular atrophy and intestinal metaplasia involve both the gastric antrum and body, whereas in autoim- mune atrophic gastritis, the disease is essentially restricted to the gastric body. Ideally, the precise location is indicated by the endoscopist, and the biopsies from different sites are submitted in separate containers. However, using spe- cial stains can help the pathologist determine the location of the biopsy fragments received. This approach is ex- emplified in case 5. Gastric atrophy and intestinal metaplasia are associated with increased gastric cancer risk, but unlike the intestinal metaplasia of Barrett syndrome, no specific recommen- dations for surveillance have been established in the Unit- ed States, although published data in other populations have suggested a benefit.23 In that study,23 patients with
  • 5. 1590 Arch Pathol Lab Med—Vol 132, October 2008 Pathologic Diagnosis of Gastritis—Sepulveda & Patil Figure 4. Autoimmune gastritis. A, The gastric biopsy location in stomach was not provided in this case (hematoxylin-eosin, original magnification ⫻20). B, Immunohistochemical stain for synaptophysin demonstrates enterochromaffin-like cell hyperplasia (original magnifi- cation ⫻20). The inset shows individual enterochromaffin-like cells staining brown, indicated by the arrow. C, Immunohistochemical stain for gastrin is negative, indicating the biopsy site to be from the gastric oxyntic mucosa (original magnification ⫻20). extensive atrophic gastritis and intestinal metaplasia had an 11% risk of gastric malignancy. Case 4 Esophagogastroduodenoscopy of a 60-year-old man shows gastritis. The pathologist needs to rule out H pylori and gastric atrophy. The gastric site of the biopsy is not specified. Figure 4, A through C, represents the histologic findings. Findings. Examination of the biopsy material results in the following pattern of answers: 1. Are there features of chronic gastritis? Yes. 2. Are there neutrophils in the mucosa? Yes. There are neutrophils in the mucosa; therefore, there is a component of active gastritis. 3. Is there Helicobacter? No. Examination with H&E stains do not reveal H pylori bacterial forms. Immunohis- tochemical stain is performed. 4. Is there atrophy? If the biopsy is from gastric oxyntic mucosa then there is atrophy, however, if the specimen is from the antrum, it may represent chronic gastritis with- out atrophy. There is no intestinal metaplasia. 5. Are special stains recommended? Yes. Immunohis- tochemical stains for synaptophysin and gastrin are per- formed. Immunohistochemical stains for synaptophysin (Figure 4, B), show a linear pattern of synaptophysin-pos- itive cells, whereas the gastrin stain is negative. Because gastrin is negative, the biopsy is not from the gastric an- trum (G cells are characteristically located in the antrum and pylorus), and therefore, it can be established that the biopsy is of oxyntic mucosa with reduced oxyntic glan- dular profiles, establishing a diagnosis of atrophy. The lin- ear arrays of synaptophysin-positive cells represent en- terochromaffin-like cell hyperplasia. Enterochromaffin- like cell hyperplasia occurs in response to hypergastri- nemia that results from hypochlorhydria associated with gastric oxyntic cell atrophy. 6. Are additional special features seen? No. 7. Is immunohistochemical stain for H pylori positive? No. Diagnosis. Gastric oxyntic mucosa with chronic active gastritis and glandular atrophy, severe. No intestinal meta- plasia is identified. No Helicobacter organisms are identi- fied. Note: These features are most suggestive of autoim- mune gastritis. AUTOIMMUNE ATROPHIC GASTRITIS This form of gastritis (reviewed in Sepulveda et al1 and Capella et al24 ) is caused by antiparietal cell and anti-in- trinsic factor antibodies and presents as a chronic gastritis with oxyntic cell injury, and glandular atrophy essentially restricted to the oxyntic mucosa of the gastric body and fundus. The histologic changes vary in different phases of the disease. During the early phase, there is multifocal infiltration of the lamina propria by mononuclear cells and eosinophils and focal T-cell lymphocyte infiltration of ox- yntic glands with glandular destruction. Focal mucous neck cell hyperplasia (pseudopyloric metaplasia), and hy- pertrophic changes of parietal cells are also observed. During the florid phase, there is increased lymphocytic inflammation, oxyntic gland atrophy, and focal intestinal metaplasia. The end stage is characterized by diffuse in- volvement of the gastric body and fundus by chronic atro- phic gastritis associated with multifocal intestinal meta- plasia. In contrast to the gastric body, the antrum is spared. Recently, a distinct form of autoimmune gastritis, characterized by atrophic pangastritis, was reported in a small group of patients with systemic autoimmune dis- orders.25 Autoimmune gastritis is a relatively rare disease but represents the most frequent cause of pernicious anemia in temperate climates. The risk of gastric adenocarcinoma was reported to be at least 2.9 times higher in patients with pernicious anemia than in the general population, and there is also an increased risk of gastric carcinoid tu- mors. Case 5 A 47-year-old woman presents with a history of celiac disease. Esophagogastroduodenoscopy was performed, with biopsy of gastric antrum. The pathologist needs to rule out H pylori. Figure 5, A and B, illustrates the histo- logic findings. Findings. Examination of the biopsy material results in the following pattern of answers: 1. Are there features of chronic gastritis? Yes. There are large numbers of intraepithelial lymphocytes. 2. Are there neutrophils in the mucosa? No. 3. Is there Helicobacter? No. Examination with H&E
  • 6. Arch Pathol Lab Med—Vol 132, October 2008 Pathologic Diagnosis of Gastritis—Sepulveda & Patil 1591 Figure 5. Lymphocytic gastritis. A, Gastric mucosal surface epithelium is studded with intraepithelial small lymphocytes (hematoxylin-eosin, original magnification ⫻20). B, Immunohistochemistry highlights nu- merous CD3-positive T lymphocytes staining dark brown (original mag- nification ⫻20). The lymphocytes predominantly infiltrate the surface and foveolar epithelium. The thick arrow points to light-brown back- ground stain found in some glandular cells. The inset shows a higher magnification of the surface epithelium containing many intraepithelial lymphocytes (original magnification ⫻40). One individual lymphocyte is indicated by the arrow. T lymphocytes stain dark brown, whereas the nucleus of the epithelial cell nuclei stain with the blue counterstain. stain does not reveal H pylori bacterial forms. Immunohis- tochemical is performed. 4. Is there atrophy? No. There is no glandular atrophy and no intestinal metaplasia. 5. What is the topography of lesions? The chronic gas- tritis involves, at minimum, the gastric antrum. 6. Are additional special features seen? Yes. The specific features in this biopsy include a characteristic intraepithe- lial lymphocytosis. Immunohistochemical stain for CD3 is positive, highlighting a population of T lymphocytes in the mucosa and, typically, many intraepithelial lympho- cytes. 7. Are special stains recommended? Yes. Immunohis- tochemical stain for H pylori, which is negative. Diagnosis. Chronic gastritis with increased intraepi- thelial T lymphocytes. No Helicobacter organisms are iden- tified. Note: These features are consistent with lympho- cytic gastritis–associated with celiac disease. LYMPHOCYTIC GASTRITIS Lymphocytic gastritis is a type of chronic gastritis char- acterized by marked infiltration of the gastric surface and foveolar epithelium by T lymphocytes and by chronic in- flammation in the lamina propria. A diagnosis can be ren- dered when 30 or more lymphocytes per 100 consecutive epithelial cells are observed, and the counts are recom- mended in biopsies from the gastric corpus. The endo- scopic pattern is, in some cases, described as varioliform gastritis. The cause of lymphocytic gastritis is usually un- known, but some cases are seen in patients with gluten- sensitive enteropathy/celiac disease and in Ménétrier disease. Smaller numbers of intraepithelial lymphocytes can also be seen in H pylori gastritis, but the diagnosis of lympho- cytic gastritis should be reserved for cases with marked intraepithelial lymphocytosis in the absence of active H pylori gastritis. Lymphocytic gastritis can be observed in children but is usually detected in late adulthood, with average age of diagnosis of 50 years. Case 6 A 75-year-old woman presents after esophagogastro- duodenoscopy. Gastric antrum shows gastritis; the pa- thologist is asked to rule out H pylori. The histologic find- ings are shown in Figure 6. Findings. Examination of the biopsy material results in the following pattern of answers: 1. Are there features of chronic gastritis? There is min- imal chronic gastritis. 2. Are there neutrophils in the mucosa? No. 3. Is there Helicobacter? No. Examination of H&E stains does not reveal H pylori bacterial forms. 4. Is there atrophy? No. There is no atrophy or intestinal metaplasia. 5. What is the topography of lesions? The chronic gas- tritis involves, at minimum, the gastric antrum. 6. Are additional special features seen? Yes. There are diagnostic special features, including foveolar hyperplasia with a corkscrew appearance of the foveolae. The foveolar epithelium shows reactive cytologic features, including re- duced cytoplasmic mucin. The lamina propria shows con- gestion and smooth muscle hyperplasia, with prominent muscularization of the most superficial mucosa. 7. Are special stains recommended? No ancillary tests are performed. Diagnosis. Gastric antral mucosa with features con- sistent with reactive gastropathy. No H pylori organisms are identified. CHRONIC, REACTIVE (CHEMICAL) GASTROPATHY Chronic reactive gastropathy (also know as chemical gastropathy) is very common in current clinical practice. The mucosal changes are usually more prominent in the prepyloric region, but they may extend to involve the ox- yntic mucosa. The usual underlying causes include chron- ic bile reflux and long-term NSAID intake. The histopath- ologic features include mucosal edema, congestion, fibro- muscular hyperplasia in the lamina propria, and foveolar hyperplasia with a corkscrew appearance in the most se- vere forms. The foveolar epithelium characteristically shows reactive nuclear features and reduction of mucin. The epithelial changes occur with little background chron- ic inflammation. However, if there is erosion of the mu- cosa, superficial neutrophils may be present. Erosive gas-
  • 7. 1592 Arch Pathol Lab Med—Vol 132, October 2008 Pathologic Diagnosis of Gastritis—Sepulveda & Patil Figure 6. Reactive gastropathy (hematoxylin-eosin, original magnifi- cation ⫻20). Figure 7. Erosive gastritis and cytomegalovirus-associated gastritis. A, Gastric mucosa with erosion in a patient with history of nonsteroidal anti-inflammatory drug use (hematoxylin-eosin, original magnification ⫻20). B, Gastric mucosal erosion with granulation tissue. Inset shows ← cytomegalovirus inclusion. This single inclusion is identified by the ar- row on the background granulation tissue (hematoxylin-eosin, original magnifications ⫻20 and ⫻40 [inset]). tritis (Figure 7, A) can present clinically as acute gastritis, often associated with NSAID intake. The features associated with bile reflux are typically found in patients with partial gastrectomy, in whom, the lesions develop near the surgical stoma. However, alter- ations induced by bile reflux also affect the intact stomach. A recent study26 reported altered mucin expression in re- active gastropathy, including aberrant expression of MUC5Ac in pyloric glands. Evaluation of mucin-expres- sion patterns can be useful to support a diagnosis of re- active gastropathy; however, additional studies are war- ranted to validate this potential application of mucin im- munohistochemistry. Case 7 A 45-year-old woman presents with a history of bone marrow transplant. Esophagogastroduodenoscopy shows gastric erosion. The histologic findings are represented in Figure 7, B. Findings. Examination of the biopsy material results in the following pattern of answers: 1. Are there features of chronic gastritis? Yes. The sam- ple of gastric mucosa reveals mucosal erosion with gran- ulation tissue and associated chronic and acute inflam- mation. 2. Are there neutrophils in the mucosa? Yes. There are superficial neutrophils in the mucosa, but they are limited to the area of mucosal erosion. 3. Is there Helicobacter? No. Examination with H&E stain does not reveal such bacterial forms. 4. Is there atrophy? No. There is no atrophy or intestinal metaplasia. 5. What is the topography of lesions? Away from the areas of erosion, there is no evidence of gastritis; therefore, the location of the biopsy is not contributory in this case. 6. Are additional special features seen? Yes. There are special features including enlarged cells, arousing suspi- cion of cytomegalovirus inclusions in the granulation tis- sue. 7. Are special stains recommended? Yes. Immunohis- tochemical stain for cytomegalovirus reveals rare but char- acteristic viral inclusions (not shown). Diagnosis. Gastric antral mucosa with erosion and cy- tomegalovirus inclusions, consistent with cytomegalovirus- associated gastritis. CYTOMEGALOVIRUS GASTRITIS Cytomegalovirus infection of the stomach is observed in patients with underlying immunosuppression. Histo- logically, intranuclear eosinophilic inclusions and smaller intracytoplasmic inclusions in enlarged cells are charac- teristic. A patchy, mild inflammatory infiltrate is observed in the lamina propria. Viral inclusions are present in en- dothelial or mesenchymal cells in the lamina propria and may be seen in gastric epithelial cells. Severe activity may result in mucosal ulceration.
  • 8. Arch Pathol Lab Med—Vol 132, October 2008 Pathologic Diagnosis of Gastritis—Sepulveda & Patil 1593 COMMENT Most types of gastritis can be diagnosed with H&E stains. To reach a determination of etiology and a specific diagnostic entity, a limited list of questions can be used to evaluate the histopathology of gastric biopsies, which can lead to a pattern of answers that corresponds to a specific diagnosis of the most common types of gastritis. Although not ideal, the diagnosis of gastritis can be reached from limited biopsy material, even when the lo- cation of the biopsy is not indicated. If the biopsy site is not known, immunohistochemical stains for synaptophy- sin and gastrin can help determine the biopsy location, permitting a specific diagnosis of atrophic gastritis type. Helicobacter pylori immunohistochemical stains can be par- ticularly useful when moderate to severe chronic gastritis or any active gastritis is present but no Helicobacter organ- isms are identified on H&E stains, when extensive intes- tinal metaplasia is present, and to evaluate follow-up bi- opsies after antibiotic treatment for H pylori. At the end of the day, there are a number of cases with a diagnosis of chronic inactive gastritis, generally mild, for which a specific etiology cannot be determined by histo- pathologic examination alone. This may be accounted for by limited tissue sampling, nonspecific focal, mild, chronic inactive gastritis associated with various systemic disor- ders, or as yet uncharacterized forms of gastritis. References 1. Sepulveda AR, Dore MP, Bazzoli F. Chronic gastritis. Available at: http:// www.emedicine.com. Accessed November 27, 2007. 2. Srivastava A, Lauwers GY. Pathology of non-infective gastritis. Histopathol- ogy. 2007;50:15–29. 3. McKenna BJ, Appelman HD. Primer: histopathology for the clinician—how to interpret biopsy information for gastritis. Nat Clin Pract Gastroenterol Hepatol. 2006;3:165–171. 4. Dixon MF, Genta RM, Yardley JH, Correa P, the participants in the Inter- national Workshop on the Histopathology of Gastritis, Houston 1994. Classifi- cation and grading of gastritis: the updated Sydney System. Am J Surg Pathol. 1996;20:1161–1181. 5. Singhal AV, Sepulveda AR. Helicobacter heilmannii gastritis: a case study with review of literature. Am J Surg Pathol. 2005;29:1537–1539. 6. Sipponen P, Kekki M, Haapakoski J, Ihamaki T, Siurala M. Gastric cancer risk in chronic atrophic gastritis: statistical calculations of cross-sectional data. Int J Cancer. 1985;35:173–177. 7. Uemura N, Okamoto S, Yamamoto S, et al. Helicobacter pylori infection and the development of gastric cancer. N Engl J Med. 2001;345:784–789. 8. Asaka M, Sugiyama T, Nobuta A. et al. Atrophic gastritis and intestinal meta- plasia in Japan: results of a large multicenter study: Helicobacter. 2001;6:294– 299. 9. Correa P, Haenszel W, Cuello C, Tannenbaum S, Archer M. A model for gastric cancer epidemiology. Lancet. 1975;2:58–60. 10. Parsonnet J, Hansen S, Rodriguez L, et al. Helicobacter pylori infection and gastric lymphoma. N Engl J Med. 1994;330:1267–1271. 11. Wotherspoon AC, Ortiz-Hidalgo C, Falzon MR, Isaacson PG. Helicobacter pylori–associated gastritis and primary B-cell gastric lymphoma. Lancet. 1991; 338:1175–1176. 12. Jaffe ES, Harris NL, Stein H, Vardiman JW. Pathology and Genetics of Tu- mours of Hematopoietic and Lymphoid Tissues. Lyon, France: IARC Press; 2001. World Health Organization Classification of Tumours; vol 3. 13. Tagkalidis P, Royce S, Macrae F, Bhathal P. Selective colonization by Hel- icobacter pylori of the deep gastric glands and intracellular canaliculi of parietal cells in the setting of chronic proton pump inhibitor use. Eur J Gastroenterol Hepatol. 2002;14:453–456. 14. Rugge M, Genta RM. Staging and grading of chronic gastritis. Hum Pathol. 2005;36:228–233. 15. Huang JQ, Sridhar S, Chen Y, Hunt RH. Meta-analysis of the relationship between Helicobacter pylori seropositivity and gastric cancer. Gastroenterology. 1998;114:1169–1179. 16. Correa P, Houghton J. Carcinogenesis of Helicobacter pylori. Gastroenter- ology. 2007;133:659–72. 17. Sepulveda AR, Coelho LG. Helicobacter pylori and gastric malignancies. Helicobacter. 2002;7(suppl 1):37–42. 18. Jonkers D, Stobberingh E, de Bruine A, Arends JW, Stockbrugger R. Eval- uation of immunohistochemistry for the detection of Helicobacter pylori in gastric mucosal biopsies. J Infect. 1997;35:149–154. 19. Toulaymat M, Marconi S, Garb J, Otis C, Nash S. Endoscopic biopsy pa- thology of Helicobacter pylori gastritis: comparison of bacterial detection by im- munohistochemistry and Genta stain. Arch Pathol Lab Med. 1999;123:778–781. 20. Xin W, Greenson JK. The clinical significance of focally enhanced gastritis. Am J Surg Pathol. 2004;28:1347–1351. 21. Price A. The Sydney System: histological division. J Gastroenterol Hepatol. 1991;6:209–222. 22. Rugge M, Correa P, Dixon MF. et al. Gastric mucosal atrophy: interob- server consistency using new criteria for classification and grading. Aliment Phar- macol Ther. 2002;16:1249–1259. 23. Whiting JL, Sigurdsson A, Rowlands DC, Hallissey MT, Fielding JW. The long term results of endoscopic surveillance of premalignant gastric lesions. Gut. 2002;50:378–381. 24. Capella R, Fiocca C, Cornaggia M. Autoimmune gastritis. In: Graham DY, Genta RM, Dixon MF, eds. Gastritis. Philadelphia, Pa: Lippincott Williams; 1999: 79–96. 25. Jevremovic D, Torbenson M, Murray JA, Burgart LJ, Abraham SC. Atrophic autoimmune pangastritis: a distinctive form of antral and fundic gastritis associ- ated with systemic autoimmune disease. Am J Surg Pathol. 2006;30:1412–1419. 26. Mino-Kenudson M, Tomita S, Lauwers GY. Mucin expression in reactive gastropathy: an immunohistochemical analysis. Arch Pathol Lab Med. 2007;131: 86–90. Archives of Pathology & Laboratory Medicine and Archives of Ophthalmology will publish a joint theme issue on ophthalmic pathology in August 2009. Articles on diagnostic procedures, path- ologic mechanistic pathways, and translational research in retinoblastoma, melanoma, lympho- ma, orbital, and adnexal tumors in ophthalmic pathology will have the best chance for consid- eration in this theme issue. Manuscripts must be submitted no later than February 1, 2009 for consideration in the joint theme issue. All submissions will undergo our usual peer review process. Important: When submitting a manuscript for this theme issue, be certain to mention this in both the cover letter and the Comment section within the AllenTrack submission system. To view our Instructions for Authors, visit http://arpa.allenpress.com/pdf/instructions㛮for㛮authors.pdf